I MBBS Regulations - 08.01.2022

You might also like

Download as docx, pdf, or txt
Download as docx, pdf, or txt
You are on page 1of 84

Dr. NTR UNIVERSITY OF HEALTH SCIENCES: A.P.

:
VIJAYAWADA - 520 008

Guidelines for the scheme of University Examinations 2019 - 20

1
Index

Page
Sl. Guidelines for the scheme of University Examinations 2019 – 20
No No.

1. Introduction to the competency based undergraduate medical education curriculum 3-4

2. Duration and details of Course 5


3. Guidelines for assessment 6-7
4. Examination schedule 8
5. Teaching Hours 9

6. Marks distribution for various subjects in University examinations 10

7. Eligibility to appear for examination, pass criteria and classification of results 11 -12
8. Introduction for Anatomy 13 -14
9. Syllabus for anatomy 15 -44
10. Distribution of Marks for Anatomy 45 -51
11. Introduction for Physiology 52 -53
12. Syllabus for Physiology 54 -61
13. Distribution of Marks for Physiology 62 -67
14. Introduction for Biochemistry 68 -69
15. Syllabus for Biochemistry 70 -81
16. Distribution of marks for Biochemistry 82 -87

2
INTRODUCTION TO THE COMPETENCY BASED UNDERGRADUATE MEDICAL EDUCATION
CURRICULUM

The revised curriculum has attempted to enunciate the competencies the student must be
imparted and should have learnt, with clearly defined teaching-learning strategies and effective
methods of assessment. These and other goals identified in the curriculum are to be implemented
in all medical colleges under the ambit of Medical Council of India from August 2019 and to
smoothen this process, guidelines have been prepared for its effective implementation.
OBJECTIVES OF MEDICAL GRADUATE TRAINING PROGRAMME:

(1) NATIONALGOALS:

At the end of undergraduate program, the medical student should be able to:

a) Recognize `health for all' as a national goal and health right of all citizens and by
undergoing training for medical profession fulfill his/her social obligations towards
realization of this goal.

b) Learn every aspect of National policies on health and devote himself/herself to its practical
implementation

c) Achieve competence in practice of holistic medicine, encompassing promotive,


preventive, curative and rehabilitative aspects of common diseases.
d) Develop scientific temper, acquire educational experience for proficiency in profession
and promote healthy living.
e) Become exemplary citizen by observation of medical ethics and fulfilling social and
professional obligations, so as to respond to national aspirations.

(2) INSTITUTIONALGOALS:

In consonance with the national goals each medical institution should evolve
institutional goals to define the kind of trained manpower (or professionals) they intend to
produce.
The undergraduate students coming out of a medical institute should:

a) Be competent in diagnosis and management of common health problems of the

3
individual and the community, commensurate with his/her position as a member of
the health team at the primary, secondary or tertiary levels, using his/her clinical
skills based on history, physical examination and relevant investigations.
b) Be competent to practice preventive, promotive, curative and rehabilitative
medicine in respect to the commonly encountered health problems.
c) Appreciate rationale for different therapeutic modalities, be familiar with the
administration of the “essential drugs" and their common side effects.
d) Be able to appreciate the socio-psychological, cultural, economic and
environmental factors affecting health and develop humane attitude towards the
patients in discharging one's professional responsibilities.
e) Possess the attitude for continued self-learning and to seek further expertise or to
pursue research in any chosen area of medicine.

4
DURATION AND DETAILS OF COURSE

The MBBS course comprises four and a half years, followed by compulsory rotatory internship of
one year. The course follows semester system, each semester consisting of six months. Normally
the MBBS course shall commence on 1st August of an academic year.

5
GUIDELINES FOR ASSESSMENT

Internal Assessment:
Internal assessment shall be based on day-to-day assessment. It shall relate to different
ways in which learners participate in learning process including assignments, preparation for
seminar, clinical case presentation, preparation of clinical case for discussion, clinical case
study/problem solving exercise, participation in project for health care in the community,
proficiency in carrying out a practical or a skill in small research project, a written test etc.

1. Regular periodic examinations shall be conducted throughout the course. There shall be

no less than three internal assessment examinations in each Preclinical in first


professional year. An average of the marks scored in the three internal assessment
examinations will be considered as the final internal assessment marks.
2. Day to day records and log book (including required skill certifications) should be given
importance in internal assessment. Internal assessment should be based on
competencies and skills.
3. Learners must secure at least 50% marks of the total marks (combined in theory and
practical/ clinical; not less than 40 % marks in theory and practical separately)
assigned for internal assessment in a particular subject in order to be eligible for
appearing at the final University examination of that
subject. Internal
assessmentmarkswillreflectasseparateheadofpassingatthesummativeexamination.The
learner should be made aware of the results of internal assessment. Each college can
build its own mechanism and the calendar of the college should show the details
regarding conduct of internal assessment.
4. Failure to meet prescribed 50% marks in internal assessment after availing remedial
measures will make the student ineligible for appearing at the final University
examination of that subject.
5. The third internal examination is the preliminary examination to be conducted on the
lines of the university examination. The students should be made aware of the results of
internal assessment. Internal assessment marks (theory/practical) will contribute for the

6
eligibility criteria for university exam. Internal assessment marks will reflect under
separate head in the marks card of the university examination.

7
University Examinations
University examinations are to be designed with a view to ascertain whether the
candidate has acquired the necessary knowledge, minimal level of skills, ethical and
professional values with clear concepts of the fundamentals which are necessary for him/her to
function effectively and appropriately as a physician of first contact. Assessment shall be
carried out on an objective basis to the extent possible.

Nature of questions will include different types such as structured essays Long Answer
Questions (LAQ), Problem based question (PBQ), Short Answers Questions (SAQ) and Multiple-
Choice Questions (MCQ).
Marks for each part should be indicated separately. MCQs shall be accorded a weightage
of not more than 20% of the total theory marks. In subjects that have two papers, the learner
must secure at least 40% marks in each of the papers with minimum 50% of marks in aggregate
(both papers together) to pass.

Practical/clinical examinations will be conducted in the laboratories or hospital wards.


The objective will be to assess proficiency and skills to conduct experiments, interpret data and
form logical conclusion. Clinical cases kept in the examination must be common conditions that
the learner may encounter as a physician of first contact in the community. Selection of rare
syndromes and disorders as examination cases is to be discouraged. Emphasis should be on
candidate’s capability to elicit history, demonstrate physical signs, write a case record, analyze
the case and develop a management plan.

Viva/oral examination should assess approach to patient management, emergencies,


attitude in all, ethical and professional values. Candidate’s skill in interpretation of common
investigative data, X rays, identification of specimens, ECG, etc. is to be also assessed.
There shall be one main examination in an academic year and a supplementary to be
held not later than 90 days after the declaration of the results of the main examination

8
EXAMINATIONS SCHEDULE

8
TEACHING HOURS

9
Marks distribution for various subjects in University examinations

10
1. SUBMISSION OF LABORATORY RECORD NOTEBOOK AND LOG BOOK
a. At the time of Internal and University Practical/ Clinical Examination each candidate
shall submit to the Examiners his/her laboratory record notebook duly certified by the
Head of the Department as a bonafide record of the work done by the candidate.
b. At the time of Internal and University Practical/ Clinical Examination each candidate
shall
submit to the Examiners his/her logbook duly certified by the concerned staff as a
bonafide record of the overall performance of the candidate.

2. ELIGIBILITY FOREXAMINATION
a) Shall have attended at least 75% of the total number of classes in theory and 80% in
practical/ Clinical, jointly to become eligible to appear for the examination in that
subject/ subjects.
b) An average of at least 50% marks of the total marks combined in theory and practicals
internal assessment is to be obtained in a particular subject (Not less than 40% each in
IA theory and practicals separately but total should be 50% and above).
c) Shall pass in all the Phase I(Pre‐Clinical) subjects, before joining the Phase II
(Para‐Clinical) subjects.
3. PASS CRITERIA
A candidate shall pass in Theory, Practical/Clinical, and Internal assessment
examinations components separately.
1. Out of Paper I and Paper II candidate must secure at least 50% marks in total, Minimum
of 40 % each in each paper each of the papers to pass.
2. For a pass in practical/ Clinical examination, a candidate shall secure not less than 50%
marks in aggregate, i.e., marks obtained in university practical /Clinical examination and
viva voce added together.
3. Candidate not securing 50% marks in aggregate in Theory or Practical examination in a
subject shall be declared to have failed in that subject and is required to appear for both
Theory and Practical again in the subsequent examination in that subject.
4. DECLARATION OF CLASS (Classification of results)
a) A candidate having appeared in all the subjects in the same examination and passed
11
that examination in the first regular attempt and secures 75% of marks or more of grand
total marks prescribed will be declared to have passed the examination with distinction.
Not appearing in an examination/partial attendance of examination shall be counted as
an attempt.
b) A candidate having appeared in all the subjects in the same examination and passed that
examination in the first attempt and secures 65% of marks or more but less than 75% of
grand total marks prescribed will be declared to have passed the examination in First Class.
c) A learner shall not be entitled to graduate after 10 years of his/her joining of the first
part of the MBBS course. Example a student admitted in august 2019 should graduate
(complete internship also) before July 31st 2029 (As per the Regulations on Graduate
Medical Education (Amendment), 2019-page no-66).
d) A maximum number of four permissible attempts would be available to clear the first
Professional University examination, whereby the first Professional course will have to
be cleared within 4 years of admission to the said course. Partial attendance at any
University examination shall be counted as an availed attempt. A student admitted in
august 2019 should have passed first-year MBBS before August 2023, during which he is
not permitted to take more than 4 permissible attempts. If the candidate does not pass
first year MBBS within four years or in four permissible attempts, he will be discharge d
from the course(As per the Regulations on Graduate Medical Education (Amendment),
2019-page no-66)
e) A candidate having appeared in all the subjects in the same examination and passed
that examination in the first attempt/ more attempts and secures 50% of marks or more
but less than 65% of grand total marks prescribed will be declared to have passed the
examination in Second Class.
Internal Assessment
For first-year MBBS, minimum three Internal Assessments to be conducted in theory and
practical, one of which should be pre-university/ Pre-final examinations. Total marks for
Internal Assessment 60, of which 25 marks for theory,25 Marks for practicals, 05(five) marks
for logbook, 05(five) marks for practical records. Eligibility to take university examination 30

out of 60 marks.

12
4. COURSE CONTENTS
ANATOMY

Human Anatomy
(a) Competencies: The undergraduate must demonstrate:
1. Understanding of the gross and microscopic structure and development of human body,
2. Comprehension of the normal regulation and integration of the functions of the organs
and systems on basis of the structure and genetic pattern.
3. Understanding of the clinical correlation of the organs and structures involved and
interpret the anatomical basis of the disease presentations.
(b) Integration: The teaching should be aligned and integrated horizontally and vertically in organ
systems with clinical correlation that will provide a context for the learner to understand the
relationship between structure and function and interpret the anatomical basis of various
clinical conditions and procedures.

The subject of anatomy deals with the study of the human body from the cellular to the
macroscopic level. The complex design of the human body will be studied in the context of
the competencies that a basic medical doctor needs to achieve by the end of the MBBS
course. Thus, the emphasis will be on clinical, functional and applied anatomy that can be
utilized by a clinician.
This draft syllabus has been created from the list of competencies mentioned in the
Competency Based Curriculum (CBC) developed by the Medical Council of India for the
First MBBS Batch of 2019-20.

Goals and department all objectives for the Under Graduate (MBBS) Curriculum

Goal
The broad goal of the anatomy curriculum is to provide a comprehensive, scientific
knowledge of the structure and development of the human body in order to
understand the anatomical basis of normal and disease presentations including patient
management.

13
Objectives:
A. Knowledge
At the end of the course student should be able to:
1. Explain the gross structure, normal disposition and integrated functions of organ
systems in order to understand the anatomical basis of common disease
presentations and clinical procedures.
2. Describe the microscopic structure of various organs and correlate their structure
with functions, in order to understand their altered state in various disease
processes.
3. Describe the basic principles behind the sequential development of organs systems as

14
a prerequisite to explaining the developmental basis of common variations and
congenital anomalies.
4. Describe the normal structure and functions of chromosome sand genesis to
understand the genetic basis of common genetic abnormalities.
B. Skills
At the end of the course the student should be able to:
1. Demonstrate the surface marking of clinically important structures in the cadaver and
correlate it with living anatomy.
2. Locate and identify tissues and cells under the light microscope.
3. Identify important structures visualized by imaging techniques, specifically
radiographs, computerized tomography (CT) scans, MRI and ultrasonography.
4. Demonstrate the various movements at the important joints in the human body.
5. Accurately palpate the pulsations of arteries at the most appropriate sites.

C. Attitude and communication skills


At the end of the course, the student should be able to:
1. Show due respect in handling human body parts and cadavers during dissection.
2. Communicate effectively with peers and teachers in small group teaching-learning
activities.
3. Demonstrate the ability to work effectively with peers in a team.
4. Demonstrate professional attributes of punctuality, accountability and respect for
teachers and peers.
5. Appreciate the issues of equity and social accountability while undergoing early clinical
exposure.

15
Syllabus

A. GENERAL ANATOMY

Topic: Anatomical terminology (AN1.1)


 Normal anatomical position
 Planes of the body
 Terms used for relations and comparison
 Terms used for movements of the body

Topic: General features of bones and Joints (AN1.2, AN2.1toAN2.6)


 Composition of bone and bone marrow
 Parts, blood and nerve supply of a long bone
 Laws of classification*
 Special features of a sesamoid bone*
 Types of cartilage with its structure and distribution in body
 Joints with subtypes and examples
 Nerve supply of joints and Hilton’s law

Topic: General features of Muscle (AN3.1 to AN3.3)


 Classification of muscle tissue according to structure and action
 Parts of skeletal muscle
 Differences between tendons and aponeuroses with examples
 Shunt and spurt muscles*

Topic: General features of skin and fascia (AN4.1 to AN4.5)


 Types of skin and dermatomes in body*
 Structure and function of skin
 Superficial fascia along with fat distribution in body
 Modifications of deep fascia with its functions

16
 Principles of skin incisions*

Topic: General features of the cardiovascular system (AN5.1 to AN5.8)


 Differences between blood vascular and lymphatic system
 Differences between pulmonary and systemic circulation
 General differences between arteries and veins
 Functional differences between elastic, muscular arteries and arterioles
 Concept of portal system with examples
 Concept of anastomoses and collateral circulation with significance of end arteries
 Functions of meta arterioles, precapillary sphincters, arterio venous anastomoses*
 Definition of thrombosis, infarction and aneurysm*
 Concept of anastomoses and collateral circulation with significance offend arteries
 Functions of meta arterioles, precapillary sphincters, arterio venous anastomoses*
 Definition of thrombosis, infarction and aneurysm*
Topic: General Features of lymphatic system (AN6.1 to AN6.3)
 Components and functions of the lymphatic system*
 Structure of lymph capillaries and mechanism of lymph circulation*
 Concept of lymphedema and spread of tumors via lymphatics and venous system*
Topic: Introduction to the nervous system(AN7.1toAN7.8)
 General plan of nervous system with components of central, peripheral and autonomic
nervous systems
 Components of nervous tissue and their functions
 Parts of a neuron
 Classification of neurons based on structure and function
 Structure of a typical spinal nerve
 Principles of sensory and motor innervation of muscles*
 Concept of loss of innervation of a muscle with its applied anatomy
 Type of synapses*
 Differences between sympathetic and spinal ganglia*

17
B. GENERALHISTOLOGY 15 Hours

Topic: Epithelium (AN65.1 to AN65.2)


 Identification of epithelium under the microscope
 Correlation of structure and function of epithelia
 Ultra structure of epithelium*

Topic: Connective tissue histology (AN66.1 to AN66.2)


 Types of connective tissue with functional correlation
 Ultra structure of connective tissue*

Topic: Muscle histology (AN67.1 to AN67.3)


 Classification of muscle
 Structure-function correlation of muscle
 Ultra structure of muscle tissue*

Topic: Nervous tissue histology (AN68.1toAN68.3)


 Description and identification of unipolar and multipolar neurons, ganglia, peripheral
nerve
 Structure-function correlation of neuron
 Ultra structure of nervous tissue*

Topic: Blood vessels – histology (AN69.1 to AN69.3)


 Identification of elastic and muscular blood vessels, capillaries under the microscope
 Types and structure-function correlation of blood vessels
 Ultra structure of blood vessels*

Topic: Glands and Lymphoid tissue (AN70.1 to AN70.2)


 Identification of exocrine glands under the microscope
 Differentiation between serous, mucous and mixed acini

18
 Identification of lymphoid tissue under the microscope
 Micro anatomy of lymph node, spleen, thymus, tonsil and correlation of structure with
function
Topic: Bone and Cartilage (AN71.1 to AN71.2)
 Identification of bone under the microscope
 Types and structure-function correlation of bone
 Identification of cartilage under the microscope
 Types and structure-function correlation of cartilage

Topic: Integumentary System (AN72.1)


 Identification of skin and its appendages under the microscope
 Correlation of structure and function

C. GENETICS 10 Hours
Topic: Chromosomes (AN73.1 to AN73.3)
 Structure of chromosomes with classification
 Technique of karyo typing with its applications
 Lyonʹs hypothesis

Topic: Patterns of Inheritance (AN74.1 to AN74.4)


 Various modes of inheritance with examples
 Pedigree charts for the various types of inheritance
 Examples of diseases of each mode of inheritance
 Multi factorial inheritance with examples
 Genetic basis and clinical features of achondroplasia, cystic fibrosis, vitamin D
resistant rickets, haemophilia, Duchenne’s muscular dystrophy and sickle cell
anaemia*
Topic: Principle of Genetics, Chromosomal Aberrations and Clinical Genetics
(AN75.1 to AN75.5)
 Structural and numerical chromosomal aberrations

18
 Mosaics and chimaeras with examples
 Genetic basis and clinical features of Prader Willi syndrome, Edward syndrome and Patio
syndrome*
 Genetic basis of variation: polymorphism and mutation
 Principles of genetic counselling

D. GENERAL EMBRYOLOGY 15 Hours

Topic: Introduction to embryology (AN76.1 TO AN76.2)


 Stages of human life
 Terms phylogeny, ontogeny, trimester, viability

Topic: Game to genesis and fertilization (AN77.1 to AN77.6)


 Uterine changes occurring during the menstrual cycle
 Synchrony between the ovarian and menstrual cycles
 Spermatogenesis and cogenesis

 Stages and consequences of fertilization


 Anatomical principles underlying contraception
 Teratogenic influences; fertility and sterility, surrogate motherhood, social significance of
“sex ratio”*
Topic: Second week of development (AN78.1 to AN78.5)
 Cleavage and formation of blast cyst
 Development of trophoblast
 Process of implantation and common abnormal sites of implantation
 Formation of extraembryonic mesoderm and coelom, bilam in ardisc and prochordal
plate
 Abortion, decidual reaction, pregnancy tests

Topic: 3rd to 8th week of development (AN79.1 to AN79.6)


 Formation and fate of the primitive streak

19
 Formation and fate of notochord
 Process of neurulation
 Development of somites and intra embryonic coelom
 Embryological basis of congenital malformations, nucleus pulposus, sacrococcygeal
teratomas, neural tubed effects
 Diagnosis of pregnancy in first trimester*
 Role of teratogens, alpha-fetoprotein*

Topic: Fetal membranes (AN80.1 to AN80.7)


 Formation, functions and fate of chorion, amnion, yolk sac,all antois and decidua
 Formation and structure of umbilical cord
 Formation of placenta, its physiological functions, foe to maternal circulation and
placental barrier
 Embryological basis of twinning in monozygotic and dizygotic twins
 Role of placental hormones in uterine growth and parturition
 Embryological basis of estimation of fetal age*
 Types of umbilical cord attachments*

Topic: Prenatal Diagnosis (AN81.1 to AN81.3)


 Methods of prenatal diagnosis
 Indications, process and disadvantages of amniocentesis
 Indications, process and disadvantages of chorion villus biopsy

E. UPPER LIMB 50 Hours

Topic: Features of individual bones (Upper Limb) (AN8.1 to AN8.6)


 Clavicle, scapula, humerus, radius, ulna side determination, anatomical position and
important features
 Joints formed by the given bone
 Peculiarities of clavicle
 Muscle group attachments on above bones

20
 Identification and naming of bones in articulated hand
 Parts of metacarpals and phalanges
 Peculiarities of pisiform
 Scaphoid fracture and basis of a vascular necrosis*

Topic: Pectoral region (AN9.1 to AN 9.3)


 Pectoralis major, pectoralis minor attachment, nerve supply and action
 Breast location, extent, deep relations, structure, age changes, blood supply, lymphatic
drainage, microanatomy and applied anatomy
 Development of breast*

Topic: Axilla, Shoulder and Scapular region (AN 10.1 to AN10.13)


 Axilla boundaries and contents
 Axillary artery and tributaries of vein origin, extent, course, parts, relations and
branches
 Brachial plexus formation, branches, relations, area of supply of branches, course and
relations of terminal branches
 Axillary lymph nodes anatomical groups and areas of drainage
 Variations information of brachial plexus
 Erb’s palsy and Klumpke’s paralysis anatomical basis and clinical features*
 Enlarged axillary lymph nodes – anatomical basis*
 Latissmus dorsi and trapezius location, attachment, nerve supply and actions
 Arterial anastomosis around the scapula*
 Boundaries of triangle of auscultation*
 Deltoid and rotator cuff muscles
 Serratus anterior attachment and actions
 Shoulder joint type, articular surfaces, capsule, synovial membrane, ligaments,
relations, movements, muscles involved, blood supply, nerve supply and applied
anatomy
 Anatomical basis of injury to axillary nerve during intra muscular injections*

21
Topic: Arm and Cubital fossa (AN11.1 to AN11.6)
 Muscle groups of upper arm
 Biceps and tricepsbrachii
 Important nerves and vessels in arm origin, course, relations, branches (or tributaries),
termination
 Venepuncture of cubital veins anatomical basis
 Saturday night paralysis anatomical basis
 Cubital fossa boundaries and contents
 Anastomosis around elbow joint*

Topic: Forearm and hand (AN12.1 to AN12.15)


 Ventral forearm muscle groups with attachments, nerve supply and actions
 Nerves and vessels of forearm origin, course, relations, branches (or tributaries),
termination
 Flexor retinaculum identification and attachments

 Anatomical basis of carpal tunnel syndrome


 Small muscles of hand
 Movements of thumb and muscles involved
 Blood vessels and nerves in hand course and branches
 Anatomical basis of claw hand
 Fibrous flexor sheaths, ulnar bursa, radial bursa and digital synovial sheaths
 Infection of facial spaces of palm*
 Dorsal forearm muscle groups, attachments, nerve supply and actions
 Origin, course, relations, branches (or tributaries), termination of important nerves and
vessels of back of forearm
 Wrist drop annatto mical basis
 Compartments deep to extensorretinaculum
 Extensor expansion – identification and formation

22
Topic: General Features, joints, radiographs and surface marking (AN13.1 to AN13.8)
 Fascia of upper limb and compartments
 Veins of upper limb
 Lymphatic drainage of upper limb
 Dermatomes of upper limb*
 Elbow joint, proximal and distal radio ulnar joints, wrist joint and first carp
metacarpal joint type
 particular surfaces, capsule, synovial membrane, ligaments, relations, movements,
blood and nerve supply
 Sternoclavicular joint, acromioclavicular joint, carpometacarpal joints and meta
carpophalangeal joints*
 Bones and joints of upper limb seen in anteroposterior and lateral view radiographs
of shoulder region, arm, elbow, forearm and hand
 Bony land marks of upper limb jugular notch, sterna angle, acromialangle, spine of
thescapula, vertebrallevel of them edial end, inferior angle of the scapula
 Surface projection of cephalic and basilicvein
 Palpation of brachial artery and radialartery
 Testing of muscles: trapezius, pectoralis major, serratus anterior, latissimus dorsi,
deltoid, biceps brachii, brachioradialis
 Development of upper limb*

F. THORAX 50 Hours

Topic: Thoracic cage (AN21.1 to AN21.11)


 Salient features of sternum, typical rib, 1st rib and typical thoracic vertebra
 Features of 2nd, 11th and 12thribs*

 Features of 1st, 11th and 12 th thoracic vertebrae*


 Boundaries of thoracic inlet, cavity and outlet
 Extent, attachments, direction of fibers, nerve supply and actions of intercostal
muscles

23
 Course, relations and branches of a typical intercostals nerve
 Origin, course and branches/ tributaries of anterior, posterior inter cost all vessels
and internal thoracic vessels

 Origin, course, relations and branches of atypical intercostal nerve, superior


intercostal artery and subcostalartery*
 Type, articular surfaces and movements of manubriosternal, costovertebral,
costotransverse and xiphisternaljoints
 Mechanics and types of respiration
 Costochondral and inter chondral joints*
 Boundaries and contents of the superior, anterior, middle and posterior mediastinum

Topic: Heart and Pericardium (AN22.1 to AN22.7)


 Pericardium subdivisions, sinuses, blood supply and nerve supply
 External and internal features of each chamber of the heart
 Origin, course and branches of coronary arteries
 Anatomical basis of ischemic heart disease
 Formation, course, tributaries and termination of coronary sinus
 Fibrous skeleton of heart
 Position and arterial supply of the conducting system of heart

Topic: Mediastinum (AN23.1 to AN23.7)


 Oesophagus external appearance, relations, blood supply, nerve supply, lymphatic
drainage and applied anatomy
 Thoracic duct extent, relations, tributaries and applied anatomy
 Origin, course, relations, tributaries and termination of superior venacava, azygos,
hemiazygos and accessory hemiazygosveins
 Branches and relations of arch of aorta and descending thoracic aorta
 Location and extent of thoracic sympathetic chain
 Description of splanchnic nerves*
 Right lymphatic duct – extent, relations and applied anatomy

24
Topic: Lungs and Trachea (AN24.1 to AN24.6, AN25.1 to AN25.6)
 Pleura – extent, recesses with their applied anatomy, blood supply, lymphatic drainage
and nerve supply
 Lungs–sidedetermination,externalfeaturesincludingrootandclinicalcorrelates
 Description of bronchopulmonary segments
 Phrenic nerve formation and distribution
 Blood supply, lymphatic drainage and nerve supply of lungs
 Extent, length, relations, blood supply, lymphatic drainage and nerve supply of trachea*
Topic: Radiological anatomy of thorax (AN25.7 and AN25.8)
 Identification of structures seen on a plain x-ray chest (PAview)
 Identification of and description in brief of a barium swallow*

Topic: Surface marking of thorax (AN25.9)


 Demonstrate surface marking of lines of pleural reflection, lung borders and fissures,
trachea, heart borders, apex beat and surface projection of valves of heart
Topic: Histology of thorax (AN25.1)
 Identification, drawing and labeling of a slide of trachea and lung

Topic: Embryology of thorax (AN25.2 to AN25.6)


 Development of pleura, lung and heart
 Fetal circulation and changes occurring at birth
 Embryological basis of: 1) atrial septal defect, 2) ventricular septal defect, 3) Fallot’s
tetralogy and 4) trachea oesophagealfistula
 Development all basis of common cardiac congenital anomalies, transposition of great
vessels, dextrocardia, patent ductus arteriosus and coarctation of aorta
 Development of aortic arch arteries, superior vena cava, inferior vena cava and
coronarysinus*

25
G. ABDOMEN ANDPELVIS 200 Hours

Topic: Anterior abdominal wall (AN44.1 to AN44.7)


 Planes (transpyloric, transtubercular, subcostal, lateral vertical), regions and quadrants of
abdomen
 Anterior abdominal wall – fascia, blood vessels and nerves
 Rectus sheath – formation, contents, linea alba and linease millenaries
 Inguinalcanalextent,boundaries,contentsofinguinalcanal,Hesselbach’striangle
 Anatomical basis of inguinal hernia
 Attachments of muscles of anterior abdominal wall
 Common abdominal incisions*
 Umbilicus position, dermatome and applied aspects*
Topic: Posterior abdominal wall (AN45.1 to AN45.3)
 Thorax Colum bar fascia
 Lumbar plexus – root value, formation and branches
 Other nerve plexuses of posterior abdominal wall*
 Major subgroups of back muscles, nerve supply and action*

Topic: Male external genitalia (AN46.1 to AN46.5)


 Tests coverings, internal structure, side determination, blood supply, nerve supply and
lymphatic drainage
 Descent of testis with its applied anatomy
 Parts of epididymis
 Penis parts, components, blood supply and lymphatic drainage
 Anatomical basis of varicocoele*
 Anatomical basis of phimosis and circumcision*
 Spermatic cord and its contents

26
Topic: Abdominal cavity (AN47.1 to AN47.14)
 Greater and lesser sac boundaries and recesses
 Naming and identification of peritoneal folds and pouches
 Anatomical basis of ascites, peritonitis and subphrenicabscess*

 Spleen anatomical position, external features, peritoneal and visceral relations, blood
supply, nerve

 supply, lymphatic drainage and applied aspects


 Anatomical basis of splenic notch, accessory spleens and Kehr’s sign*
 Coeliac trunk origin, course, important relations and branches
 Abdominal part of oesophagus anatomical position, blood supply, nerve supply,
lymphatic drainage and applied aspects
 Stomach anatomical position, external features, peritoneal and visceral relations, blood
supply, nerve supply, lymphatic drainage and applied anatomy
 Anatomical basis of lymphatic spread in carcinoma stomach and different types of
vagotomy*
 Mesentery – extent, borders, contents, relations and applied aspects
 Small Intestine parts, macroscopic difference between jejunum and ileum, nerve supply
and lymphatic drainage
 Superior mesenteric artery origin, course, termination, important relations and branches
 Large intestine features, extent, peritoneal and other relations
 Caecum anatomical position, external and internal features, important peritoneal and
other relations, blood supply, nerve supply, lymphatic drainage and applied aspects
 Vermin or appendix anatomical position, external and internal features, important
peritoneal and other relations, blood supply, nerve supply, lymphatic drainage and
applied aspects
 Inferior mesenteric artery origin, course, important relations and branches
 Duodenum anatomical position, external and internal features, important peritoneal
and other relations, blood supply, nerve supply, lymphatic drainage and applied
aspects

27
 Pancreas anatomical position, external and internal features, important peritoneal
and other relations, blood supply, nerve supply, lymphatic drainage and applied
aspects
 Liver and extra hepaticbiliary apparatus anatomical position, external features,
important peritoneal relations and visceral relations, blood supply, nerve supply,
lymphatic drainage and applied aspects
 Clinical importance of Calot’s triangle*
 Anatomical basis of site of needle puncture in liver biopsy, referred pain in
cholecystitis and obstructive jaundice*
 Portal vein – formation, course, relations, tributaries and sites of porta systemic
anastomoses
 Anatomical basis of haematemesis and caput medusae in portal hypertension
 Kidneys anatomical position, side determination, coverings, external features,
important visceral relations, blood supply, nerve supply, lymphatic drainage and
applied anatomy
 Anatomical basis of radiating pain of kidney togroin*
 Ureter – extent, parts, course, relations, constrictions, blood supply, nerve supply,
lymphatic drainage and applied aspects

 Suprarenal gland anatomical position, coverings, external features, important visce ral
and other relations, blood supply, nerve supply, lymphatic drainage and applied
aspects
 Thoraco abdominal diaphragm – attachments, major and minor openings, nerve
supply and actions
 Thoraco abdominal diaphragm abnormal openings and diaphragmatic hernia*
 Abdominal aorta origin, course, important relations and branches.
 Inferior vena cava formation, course, relations and tributaries.

Topic: Pelvic wall and viscera (AN48.1 to AN48.8)


 Muscles of pelvic diaphragm

28
 Position, features, important peritoneal and other relations, blood supply, nerve
supply, lymphatic drainage and clinical aspects of important male and female pelvic
viscera
 Origin, course, important relations and branches of internal iliacartery
 Branches of sacral plexus
 Anatomical basis of suprapubic cystostomy, urinary obstruction in benign prostatic
hypertrophy, retroverted uterus, prolapse uterus, internal and external
haemorrhoids,analfistula,vasectomy,tubalpregnancyandtuballigation*
 Neurological basis of automatic bladder*
 Lobes involved in benign prostatic hypertrophy and prostate cancer*
 Structures palpable during vaginal and rectal examination*

Topic: Perineum (AN49.1 to AN49.5)


 Boundaries and contents of superficial and deep perinea pouch
 Perinea body identification and description
 Perineal membrane in male and female
 Ischiorectal fossa boundaries, contents and applied anatomy
 Anatomical basis of perineal tear, episiotomy, perianal abscess and anal fissure*

Topic: Vertebral column (AN50.1 to AN50.4)


 Curvatures of the vertebral column
 Type,articularends,ligamentsandmovementsofintervertebraljoints,sacroiliac joints and
pubic symphysis
 Site, direction of the needle and structures pierced during lumb arpuncture
 Anatomical basis of scoliosis, lord sis, prolapsed disc, spindly list hesis and spina bifida*

Topic: Sectional Anatomy of Abdomen and Pelvis (AN51.1, AN51.2)


 Cross sections at T8, T10 and L1 (transpyloric plane)levels
 Midsagittal section of male and female pelvis

29
Topic: Histology and embryology (AN52.1 to AN52.8)
 Microstructureofoesophagus,cardiooesophagealjunction*,fundusofstomach, pylorus
of stomach
 Microstructure of duodenum, jejunum, ileum

 Microstructure of colon, appendix


 Microstructure of liver, gallbladder, pancreas
 Microstructure of kidney, urethra, suprarenal gland
 Microstructure of testis, epididymis, vas deferens, penis, prostate gland
 Microstructureofovary,uterus,uterinetube,cervix*,placenta,umbilicalcord, corpusluteum*
 Development of anterior abdominal wall*
 Development and congenital anomalies of diaphragm
 Development and congenital anomalies off oregut
 Development and congenital anomalies of midgut
 Development and congenital anomalies of hindgut
 Development of urinary system
 Development of male reproductive system
 Development of female reproductive system

Topic: Osteology (AN53.1 to AN53.4)


 Lumbar vertebrae anatomical position, salient features, articulations and
attachments of muscle groups
 Sacrum and coccyx anatomical position, salient features, articulations and
attachments of muscle groups
 Bonypelvisanatomicalposition,boundariesofpelvicinlet,pelviccavityandpelvic outlet,
 True and false pelvis with sex differences
 Clinical importance sacralization of lumbar vertebra, lumbarization of 1st sacral
vertebra, types of bonypelvis*
Topic: Radiological anatomy (AN 54.1 to AN54.3)
 Features of plain X rayabdomen

30
 Contrast X ray barium swallow, barium meal, bariumenema
 Cholecysto graphy
 Intra venous pyelography
 Hysterosalpingo graphy
 ERCP*
 CT abdomen*
 MRI abdomen and pelvis*
 Abdominal arterio graphy*

Topic: Surface marking (AN 55.1 and AN55.2)


 Regions and planes of abdomen
 Superficial inguinal ring
 Deep inguinal ring
 McBurney’s point
 Renalangle
 Murphy’s point

 Surface projections of stomach, liver, fundus of gall bladder, spleen, duodenum,


pancreas, ileocaecal junction, kidneys and root of mesentery, abdominal aorta and
inferior venacava
H. LOWER LIMB 90 Hours

Topic: Features of individual bones (lower limb) (AN 14.1 – 14.4)


 Hip bone, femur, patella, tibia, fibula side determination, anatomical position and
important features
 Joints formed by the given bone
 Muscle group attachments on above bones
 Importance of ossification of lower end of femur and upper end of tibia
 Identification and naming of bones in articulated foot with individual muscle
attachments*

31
Topic: Front & Medial Side of Thigh (AN15.1 to AN15.6)
 Origin, course, relations, branches (or tributaries), termination of important nerves
and vessels of anterior high
 Major muscles with their attachment, nerve supply and actions
 Femoral triangle boundaries and contents
 Anatomical basis of psoas abscess & femoral hernia*
 Adductor canal – boundaries and contents

Topic: Gluteal region & Back of thigh (AN16.1 to AN16.6)


 Origin, course, relations, branches (or tributaries), termination of important nerves
and vessels of gluteal region
 Major muscles with their attachment, nerve supply and actions
 Anatomical basis of sciatic nerve injury during gluteal intra muscular injections
 Anatomical basis of Trendelen burgsign
 Hamstring group of muscles with their attachment, nerve supply and actions
 Origin, course, relations, branches (or tributaries), termination of important nerves
and vessels on the back of thigh
 Popliteal fossa boundaries, roof, floor, contents and relations

Topic: Hip joint (AN17.1 to AN17.3)


 Type, articular surfaces, capsule, synovial membrane, ligaments, relations,
movements and muscles involved, blood and nerve supply, bursae around the hip
joint
 Anatomical basis of complications of fracture neck offemur*
 Dislocation of hip joint and surgical hipreplacement*
Topic: Knee joint, Antero‐lateral compartment of leg & Dorsum of foot
(AN18.1 to AN18.7)
 Major muscles of anterolateral compartment of leg with their attachment, nerve supply
and actions

32
 Origin, course, relations, branches (or tributaries), termination of important nerves
and vessels of anterolateral compartment of leg Anatomical basis of footdrop
 Type, articular surfaces, capsule, synovial membrane, ligaments, relations,
movements and muscles involved, blood and nerve supply, bursae around the knee
joint
 Anatomical basis of locking and unlocking of the knee joint
 Anatomical basis of knee joint injuries*
 Anatomical basis of osteoarthritis*
Topic: Back of leg & Sole (AN19.1 to AN19.7)
 Major muscles of back of leg with their attachment, nerve supply and actions
 Origin, course, relations, branches (or tributaries), termination of important nerves
and vessels of back of leg
 Concept of “peripheral heart”
 Sole layers, muscles, vessels and nerves
 Anatomical basis of rupture of calcanealtendon*
 Factors maintaining arches of the foot and their importance
 Anatomical basis of flat foot and clubfoot*
 Anatomical basis of meta tarsalgia and plant arfasciitis*

Topic: General features, joints, radiographs & surface marking (AN 20.1 – 20.10)
 Tibiofibular and ankle joints type, articular surfaces, capsule, synovial membrane,
ligaments,relations,movementsandmusclesinvolved,bloodandnervesupply
 Subtalar and transverse tarsal joints*
 Fascia lata, venous drainage, lymphatic drainage, retinacula and dermatomes of lower
limb
 Anatomical basis of enlarged inguinal lymph nodes*
 Anatomical basis of varicose veins and deep vein thrombosis
 Bones and joints of lower limb seen in anteroposterior and lateral view radiographs
of various regions of lower limb
 Important bony landmarks of lower limb vertebral level of highest point on iliac

33
crest, anterior and posterior superior iliac spines, iliac tuberosity, pubic tubercle,
ischial tuberosity, adductor tubercle, tibial tuberosity, head of fibula, medial and
lateral malleoli, condyles of femur and tibia, sustentaculum tali, tuberosity of fifth
metatarsal and tuberosity of thenavicular
 Palpation of arterial pulses in a simulated environment femoral, popliteal, anterior
tibial, posterior tibial and dorsalispedis
 Surface marking mid inguinal point, saphenous opening, great and small saphenous
veins, femoral nerve, sciatic, tibial, common peroneal and deep peroneal nerve
 Basic concept of development of lowerlimb*

HEAD AND NECK 200 Hours


Topic: Skull osteology (AN26.1 to AN26.7)
 Anatomical position ofskull
 Identification and naming of individual skull bones
 Features of Norma frontalis, verticalis, occipitalis, lateralis andbasalis
 Cranial cavity subdivisions, foramina and structures passing through them
 Morphological features of mandible
 Features of typical and atypical cervical vertebrae (atlas and axis)
 Concept of membranous ossification*
 Features of the 7th cervical vertebra*
Topic: Scalp (AN27.1 and AN27.2)
 Scalp layers, blood supply, nerve supply and surgical importance
 Emissary veins and their role in spread of infection from extra cranial route to
intracranial venous sinuses

Topic: Face and parotid region (AN28.1 to AN28.10)


 Muscles of facial expression and their nerve supply
 Sensory innervations of face
 Origin / formation, course, branches / tributaries of facial vessels
 Branches of facial nerve with distribution

34
 Cervical lymph nodes and lymphatic drainage of head, face and neck
 Superficial muscles of face, their nerve supply and actions
 Anatomical basis of facial nerve palsy
 Surgical importance of deep facial vein
 Parotid gland parts, borders, surfaces, contents, relations, nerve supply, course of its
duct and surgical importance
 Anatomical basis of Frey’s syndrome*

Topic: Posterior triangle of neck (AN29.1 to AN29.4)


 Sternocleidom as toid attachments, nerve supply, relations and actions
 Anatomical basis of Erb’s and Klumpke’s palsy
 Anatomical basis of wryneck*
 Attachments of inferior belly of omohyoid, scalenus anterior, scalenus medius and
levatorscapulae*
Topic: Cranial cavity (AN30.1 to AN30.5)
 Cranial fossae and related structures
 Major foramina with structures passing through them
 Identification and description of dural folds and dural venous sinuses
 Clinical importance of dural venous sinuses
 Effect of pituitary tumours on visual pathway*

Topic: Orbit (AN31.1 to AN31.5)


 Extraocular muscles – demonstration and description

 Nerves and vessels in the orbit demonstration and description


 Anatomical basis of Horner’ssyndrome*
 Components of lacrimal apparatus
 Anatomical basis of oculomotor, trochlear and abducent nerve palsies along with
strabismus

35
Topic: Anterior triangle of neck (AN32.1 and AN32.2)
 Boundaries and subdivisions of anteriortriangle
 Boundaries and contents of muscular, carotid, digastric and submentaltriangles

Topic: Temporal and infratemporal region (AN33.1 to AN33.5)


 Temporal and infratemporal fossae extent, boundaries andcontents
 Musclesofmasticationattachments,directionoffibres,nervesupplyandactions
 Temporomandibular joint articulating surface, type and movements
 Clinical significance of pterygoid venousplexus
 Features of dislocation of temporomandibular joint*
Topic: Submandibular region (AN34.1 and AN34.2)
 Sub mandibular salivary gland morphology, relations and nerve supply including sub
mandibular ganglion
 Anatomical basis of formation of sub mandibular stones*

Topic: Deep structures in the neck (AN35.1 to AN35.10)


 Deep cervical fascia parts, extent, attachments and modifications
 Thyroid gland location, parts, borders, surfaces, relations and blood supply
 Subclavian artery origin, parts, course and branches
 Internal jugular and brachioc ephalic veins formation, course, relations, tributaries and
termination
 Cervical lymph nodes extent, drainage and applied anatomy
 Cervical sympathetic chain extent, formation, relation and branches
 IX, X, XI and XII cranial nerves course and branches in the neck
 Anatomical basic of clinical features of thyroids wellings*
 Anatomical basis of clinical features of compression of subclavian artery and lower
trunk of brachial plexus by cervicalrib*
 Fascial spaces of neck*

36
Topic: Mouth, pharynx and palate (AN36.1 to AN36.5)
 Palatine tonsil morphology, relations, blood supply and applied anatomy
 Composition of softpalate
 Waldeyer’s lymphatic ring components and functions
 Pyriform fossa boundaries and clinical significance*
 Anatomical basis of tonsillitis, tonsillectomy, adenoids and peri tonsillarabscess*
 Clinical significance of Killian’sdehiscence*

Topic: Cavity of nose (AN37.1 to AN37.3)


 Nasalseptumandlateralwallofnose–features,bloodsupplyandnervesupply
 Paranasal sinuses location and functional anatomy
 Anatomical basis of sinusitis and maxillary sinus tumours*

Topic: Larynx (AN38.1 to AN38.3)


 Larynx morphology, structure of the walls, nerve supply, blood supply and actions of
intrinsic and extrinsic muscles
 Anatomical aspects of laryngitis*
 Anatomical basis of recurrent laryngeal nerve injury*

Topic: Tongue (AN39.1 and AN39.2)


 Tongue morphology, nerve supply, embryological basis of nerve supply, blood supply,
lymphatic drainage and actions of extrinsic and intrinsic muscles
 Anatomical basis of hypoglossal nerve palsy*

Topic: Organs of hearing and equilibrium (AN40.1 to AN40.5)


 External ear parts, blood supply and nerve supply
 Middle ear and auditory tube boundaries, contents, relations and functional anatomy
 Features of internalear*
 Anatomical basis of otitis externa and otitismedia*
 Anatomical basis of myringotomy*

37
Topic: Eyeball (AN41.1 to AN41.3)
 Eyeball parts and layers
 Anatomical aspects of cataract, glaucoma and central retinal arteryocclusion*
 Intraocular muscles position, nerve supply and actions*

Topic: Back region (AN42.1 to AN42.3)


 Contents of the vertebral canal
 Sub occipital triangle boundaries and contents
 Semis pinalis capitis and splenius capitis position, direction of fibres, relations, nerve
supply and actions*
Topic: Head and neck joints, histology, development, radiography and surface
marking (AN43.1 to AN43.9)
 Atlantooccipital joint and atlantoaxial joint movements with muscles producing them
 Microanatomyofpituitarygland,thyroidgland,parathyroidgland,tongue,salivary glands,
tonsil, epiglottis, cornea and retina
 Microanatomy of olfactory epithelium, eyelid, lip, sclera corneal junction, optic nerve,
cochlea, organ of Corti and pinealgland*
 Development and developmental basis of congenital anomalies of face, palate, tongue,
branchial apparatus, pituitary gland, thyroid gland andeye

 Testing of muscles of facial expression, extraocular muscles and muscles of mastication,


 Palpation of arteries carotid, facial and superficial temporalarteries

 Location of hyoid bone, thyroid cartilage and cricoid cartilage with their vertebral
levels
 Surface marking thyroid gland, parotid gland and duct, pterion, common carotid
artery, internal jugular vein, subclavian vein, external jugular vein, facial artery in the
face and access o rynerve
 Identify the anatomical structures in 1) Plain X-ray skull – AP and lateral
view;2)PlainX-raycervicalspineAPandlateralview;3)PlainX-rayofparanasalsinuses
 Carotid and vertebral angiograms anatomical route and anatomical structures*

38
I. NEUROANATOMY 60 Hours
Topic: Meninges and CSF (AN56.1 and AN56.2)
 Meninges layers with their extent and modifications
 Circulation of CSF with its applied anatomy

Topic: Spinal cord (AN57.1 to AN57.5)


 Spinalcordexternalfeatures,extentinchildandadultwithitsclinicalimplications
 Transverse section of spinal cord at mid cervical and mid thoracic level
 Ascending and descending tracts at mid thoracic level of spinal cord
 Anatomical basis of syringomyelia*

Topic: Medulla oblongata (AN58.1 to AN58.4)


 Medulla oblongata external features
 Transverse section of medulla oblongata at the level of 1) pyramidal decussation; 2)
sensory decussation; 3) inferior olivarynucleus
 Cranial nerve nuclei in medulla oblongata with their functional components
 Anatomical basis and effects of medial and lateral medullary syndrome*

Topic: Pons (AN59.1 to AN59.3)


 Pons external features
 Transverse section of pons at the upper and lower level
 Cranial nerve nuclei in pons with their functional components

Topic: Cerebellum (AN60.1 to AN60.3)


 Cerebellum external and internal features
 Connections of cerebellar cortex and intra cerebellarnuclei
 Anatomical basis of cerebellardys function*

Topic: Midbrain (AN61.1 to AN61.3)


 Midbrain external and internal features
 Internal features of midbrain at the level of superior and inferior colliculus
 Anatomical basis and effects of Benedikt’s and Weber’s syndrome*

Topic: Cranial nerve nuclei and cerebral hemispheres (AN62.1 to AN62.6)


 Cranial nerve nuclei with their functional components
 Cerebral hemispheres – poles, surfaces, sulci, gyri and functional areas
 The white matter ofthe cerebrum
 Basal ganglia and limbic lobe parts and major connections
 Dorsal thalamus, hypothalamus, epithalamus, metathalamus and subthalamus
 Boundaries, parts, gross relations, major nuclei and connections
 Circle of Willis formation, branches and major areas of distribution

Topic: Ventricular system (AN63.1 and AN63.2)

 Lateral, 3rd and 4th and ventricles parts, boundaries and features
 Anatomical basis of congenital hydrocephalus*

Topic: Histology and Embryology (AN64.1 to AN64.3)


 Micro anatomical features of spinal cord, cerebellum, and cerebrum
 Development of neural tube, spinal cord, medulla oblongata, pons, midbrain,
cerebral hemispheres, and cerebellum
 Various types of open neural tube defects with their embryological basis*

J. ETHICS IN ANATOMY –AN82.1 4 Hours


 Demonstrate respect and follow the correct procedure when handling cadavers and
other biologic tissue
Summary of tim e allotted, teaching and learning methods and student
assessment
TIME ALLOTTED

Curricular component Time allotted in


hours
Lectures 220
Small group teaching / tutorials / integrated learning 415
/practical
Self‐directed learning 40
Early clinical exposure (basic science correlation and 30 (18 +12)
clinical
skills)
AETCOM module 5 4
Formative assessment and term examinations 30
Total 739

To be noted:
 The number of hours mentioned a bove are rough guidelines that can be
modified to suit the specific requirements of a medical college.
 It is recommended that did actic teaching be restricted to less than one third of
the total time allotted for that discipline.
 Greater emphasis is to be laid on hands-on training, symposia, seminars, small
group discussions, problem-oriented and problem-based discussions and self-
directed learning.
 Students must been courage to taken active part in and shared responsibility for
their learning.
Suggested guidelines for the teaching and learning methods

Lectures
 All lectures to have well defined specific learning objectives which are linked to the
relevant competencies. Learning objectives should be observable and assessable.
Bloom’s taxonomy can be used as a reference in choosing verbs for defining the
learning objectives.
 The focus should be on the must know component of the topic.
 As anatomy is a largely visually based subject appropriate pictures and video scan be
utilized.
 The anatomical basis of clinical conditions per training to the topic to be addressed.
 Interactivity needs to be built into the lecture by asking open-ended questions,
quizzes, incomplete handouts, creation of models, solving problems or a flipped-
classroom approach, to name a few methods.

Other methods
 Team based learning can be used in place of did actic lectures.
 Case based learning can be used for tutorials.
 Seminars and assignments will encourage active learning by the students.
Dissection
 All dissections to have specific learning objectives which are linked to the relevant
competencies and are clinically relevant.
 The focus should be on identifying and how to identify important structures of the
region being dissected.
 Students should be encouraged to perform the dissections using relevant resources
like a good dissection manual and dissection videos, with faculty as facilitators.
 The dissection can be accompanied by relevant surface anatomy exercises,
demonstration of suitable radiological images and analysis of joint movements.
 The students should be encouraged to observe the dissec tion of cadavers in
neighboring tables so that they appreciate common anatomical variations.
 Each dissection can be accompanied by suitable clinical case scenarios, which can be
discussed at the end of the dissection to bring out its clinical relevance.
 Each dissection session is an excellent opportunity to reiterate the concepts of
respect for cadaver and professionalism.
Histology practical
 All histology sessions to have specific learning objectives which are linked to the
relevant competencies and are clinically relevant.
 The focus should be identifying and how to identify important structures in the
sections being viewed.
 Students should be encouraged to independently identify the salient features of the
section with faculty as facilitators.
 Each session can be accompanied by suitable clinical case scenarios which can be
discussed at the end of the session to bring out its clinical relevance.
 Each session is a good opportunity to reiterate the concept of professionalism.

Osteology
 All sessions to have specific learning objectives which are linked to the relevant
competencies and are clinically relevant.
 The focus should be identifying important structures of the bone being studied, the
joints formed by the bone and analysis of movement to occurring at these joints.
 Students should be encouraged to independently identify the salient features of the
bone being studied with faculty as facilitators.
 The session can be accompanied by the demonstration of suitable radio logical
images.
 Each session can be accompanied by suitable clinical case scenarios which can be
discussed at the end of the session to bring out its clinical relevance.
 Each session is a good opportunity to reiterate the concept of professionalism.
LIST OF RECOMMENDED BOOKS

General anatomy
• Handbook General Anatomy, Vishram Singh

Histology
• Textbook of Human Histology with colour Atlas, Inderbir Singh Embryology
• Textbook of Human Embryology, Inderbir Singh / Langmanʹsʹs textbook of Medical
Embryology, TWSadler

Human genetics
• Human Genetics, SD Gangane / Medical Genetics, GP Pal

Gross anatomy including neuroanatomy


• Cunningham’s Manual of Practical Anatomy Volumes I,II andIII
• BDChaurasia’s/Dutta’s/VishramSingh’s Textbook of Anatomy–all volumes
• Grayʹs Anatomy for Students, South Asia Edition
• Textbook of Neuro anatomy, IB Singh / Textbook of Clinical Neuro anatomy, Vishram
Singh

Surface and radiological anatomy


• Surface and radiological anatomy, A Halim

Reference book : diFiore’s Atlas of Human Histology with Functional Correlation, Victor P
Eroschenko
• Grant’s atlas / McMinn’s atlas / Netter’satlas
• Clinical Neuroanatomy, Richard Snell
• Stedman’s MedicalDictionary
• Grayʹs Anatomy ‐ The Anatomical Basis of ClinicalPractice
Please note: It is suggested that students use the latest editions of the above books.
Paper –

Scheme of University examination- I MBBS


Theory Max Marks Practicals Max Marks

Paper -1 100 Practical 80


exam
Paper -2 100 Viva 20

Total 200 Total 100

45
Paper –
1) Multiple choice questions 20X1=20M
General anatomy- 3 M
General Histology- 3M
Upper limb— 5 M Head
and neck- 5 M
Neuroanatomy- 4 M
2) Describe shoulder joint under following (1+3+4+3+4)
a. Type of joint
b. Articular surfaces
c. Ligaments
d. Rotaor cuff
e. Movements
3) Describe the thyroid gland under (4+4+4+3)
a. External features
b. Relations
c. Blood supply
d. Applied aspects
Short essays:
4) Microscopic structure of Bone
5) Fibrous joints
6) Supinators
7) Anatomical basis of carpal tunnel syndrome
8) Corpus callosum
9) Blood supply and nerve supply of scalp
10) Draw a neat labelled diagram of Spinal cord with tracts
11) A female patient of age 54 yrs came to opd complaing profound
swaeting on the while eating on history taking had a surgery on the
parotid gland recently. Basing on this answr the following (1+2+2)
a) Name the clinical condition
b) What is its anatomical basis
c) Secretomotor pathway of parotid gland
12) Inferior horn of lateral ventricle

46
Paper-II

1) MULTIPLE CHOICE QUESTIONS 20X 1= 20M


Systemic histology- 2M
Systemic embryology-2M
Genetics. -2M
Lower limb—— 4M
Abomen and pelvis- 6M
Thorax—— 4M

2) Describe in detail about stomach under 2+4+4+2+3


A. Parts
B. Relations
C.Blood supply
D. Nerve supply
E. Lymphatic drainage
3) Describe knee joint under 1+3+4+4+3
a) Type
b) Articulating surface
c) Ligaments
d) Movements
e) Locking and unlocking

Short essays
4) Microscopic structure of Testes
5) Development of pancreas
6) Karyotyping
7) Medial longitudinal arch of foot
8) A male patient aged 50 yrs came to sugicalopd with engorgemt of veins
on the legsand ulcerating wound. Trendelenburgs test was positive on
examination. Based on this answer the following 2+1+2
a. What is the clinical condition
b. What is Trendelenburg’s test
c. Name the perforators.
9) supports of Uterus
10) Describe Rt . Atrium

47
11) A male patient of age 25 yrs came to tgeopd with complaints of fever,
vomitings, pain abdomen. On examination gaurding in rt . Iliac
fossa.and psoas test positive

Based on this clinical scenario answer the following


a. What is the stuctureinvoled?
b. What is MC Burneyspoont?
c. Blood supply
12. Sinuses of pericardium
13.inguinal hernia

48
Distribution of marks – Anatomy
Paper – I

Blueprint for the anatomy theory examinations

Arteries

Muscles

Marks
Organs
Nerves

Joints
Bones
Space
Veins

Connecti

tissue
ve e
TOPIC MCQ’s
General 5
anatomy
3

General 5
histology 3

General 5
0
embryology

Upper limb 25 5

Head and neck 25 5

Neuro anatomy 15 4

TOTAL 80 Marks 20Marks


P a pe r 2

Connectiv e tissue
Arteries

Muscles
Nerves

Organs
Bones
Space

Joints

Marks
Veins

Topic MCQ’s
Systemic 5
Histology 2

Systemic 5
Embryology 2

Genetics 5 2

Lower limb 25 4

Abdomen and 25 6
Pelvis

Thorax 15 4

TOTAL 80 Marks 20 Marks

50
PHYSIOLO
GY

(a) Competencies: The undergraduates must demonstrate:


1. Understanding of the normal functioning of the organs and organ systems of the body,
2. Comprehension of the normal structure and organization of the organs and systems on
basis of the functions,
3. Understanding of age-related physiological changes in the organ functions that reflect
normal growth and development,
4. Understand the physiological basis of diseases.
(b) Integration: The teaching should be aligned and integrated horizontally and vertically in
organ systems in order to provide a context in which normal function can be correlated both
with structure and with the biological basis, its clinical features, diagnosis and therapy.

Goal:

The broad goal of the teaching of undergraduate students in Physiology aims


at providing the student comprehensive knowledge of the normal functions
of the organ systems of the body to facilitate an understanding of the
physiological basis of health and disease.

Objectives

A. Knowledge
At the end of the course the student will be able to:
1. Explain the normal functioning of all the organ systems and their interactions

for well coordinated total body function;


2. Assesstherelativecontributionofeachorgansystemtothemaintenanceofthe
milieu interior;
3. Elucidate the physiological aspects of normal growth and development;

4. Describe the physiological response and adaptations to environmental stresses;

5. List the physiological principles underlying pathogenesis and treatment of disease

B. Skills At the end of the course the student shall be able to:
At the end of the course the student will be able to:
1. Conduct experiments designed for study of physiological phenomena;

51
2. Interpret experimental/investigative data;
3. Distinguish between normal abnormal data derived as a result of tests,
which he/she has performed and observed in thelaboratory.

C. Attitude and communication skills:


At the end of the course the student will be able to:
1. Show due respect to persons who volunteer to be examined for the purpose
of learning clinical examination.
2. Communicate effectively with peers and teachers
3. Demonstrate the ability of teamwork

D. Integration:
At the end of the integrated teaching the student should acquire an integrated
knowledge of organ structure and function and the regulate or mechanisms.
List of systems included in Physiology:
 General Physiology
 Hematology
 Nerve Muscle Physiology
 Gastro Intestinal Physiology
 Cardiovascular physiology
 Respiratory physiology
 Renal Physiology
 Endocrine Physiology
 Reproductive Physiology
 Neurophysiology (Central Nervous System and Special Senses)
 Integrated Physiology

52
Syllabus for theory

General Physiology (PY1.11.9) (8hrs)

Structure and functions of a mammalian cell; Homeostasis, Intercellular


communication; Apoptosis; Transport mechanisms across cell membranes; Fluid
compartments of the body; pH & Buffer systems in the body; Evaluation of
functions of the cells and products in clinical care and research.

Hematology: (PY 2.1 – 2.13) (16 hrs)


Components of blood: formation, regulation and functions; plasma proteins –
origin, types, variations and functions; Hemoglobin synthesis, variants, functions
and its breakdown & Jaundice; Blood indices; Anemia and its classification;
Hemostasis: mechanism, regulation& disorders Anticoagulants; Blood groups,
blood banking and transfusion; Immunity: types, mechanism & regulation; ESR;
Lymph composition, circulation and functions
Nerve & Muscle Physiology: (PY 3.1 – 3.18) (12hrs)
Neuron and neuroglia: structures, types, functions; Resting membrane
potential; Action potential in nerve, skeletal & smooth muscle; Nerve fibres:
classification, functions & properties; nerve injuries, degeneration and
regeneration in peripheral nerve; Neuromuscular junction: structure, transmission
of impulses, neuromuscular blocking agents, Myasthenia gravis; Muscle fibres:
structure, types & functions; Muscle contraction; molecular basis (skeletal,
smooth), Isotonic Vs. Isometric, Energy sources and metabolism, gradation of
muscle activity; muscle dystrophy, Myopathies; Strength duration curve
Gastrointestinal Physiology :(PY4.14.10) (10hrs)
Functional anatomy and broad functions of digestive system, enteric nervous
system; GI Secretions composition, mechanism of secretion, functions, and
regulation of saliva, gastric, pancreatic, intestinal juices and bile secretion; GI
movements types, regulation, functions, reflexes; role of dietary fibres; Digestion
and absorption of nutrients; GI hormones source, regulation, functions; Gut brain
axis; structure and functions of liver and gall bladder; gastric function tests,
pancreatic exocrine function tests & liver function tests, Path physiology Achalasia
cardia, peptic ulcer, gastro oesophageal reflux disease, vomiting, diarrhoe

53
constipation, Adynamic ileus, Hirschsprungʹs disease.

Cardiovascular Physiology: (PY 5.1 – 5.16) (23 hrs)


Functional anatomy of heart; Pacemaker tissue and conducting system
generation, conduction of cardiac impulse; Properties of cardiac muscle; Cardiac
cycle; ECG recording, normal ECG, uses, cardiac axis, Abnormal ECG in common
arrhythmias, changes with hypertrophy & MI; Haemo dynamics; Heart rate factors
affecting, regulation; Cardiac output factors, regulation, measurement; Blood
pressure components, determinants, factors, regulation and applied aspect,
Regional circulation auto regulation, microcirculation, lymphatic circulation,
coronary, cerebral, capillary, skin, fetal, pulmonary and splanchnic circulation; Path
physiology shock, syncope, heart failure & coronary artery disease
Respiratory Physiology: (PY 6.1 – 6.10) (14 hrs)
Functional anatomy of respiratory tract, dead space; Mechanics of
respiration; Pressure volume changes during ventilation; Lung volume and
capacities; Alveolar surface tension; Compliance; Airway resistance; alveolar
ventilation, V/P ratio; Diffusion capacity of lungs;
TransportofrespiratorygasesOxygenandCarbondioxide;Neuralandchemicalregulatio
n of respiration; Physiology of high altitude and deep sea diving; Principles of
artificial respiration, oxygen therapy; Path physiology of dyspnoea, hypoxia,
cyanosis, asphyxia, drowning, periodic breathing; Lung function tests & its clinical
significance

Renal Physiology: (PY 7.1 – 7.9) (12 hrs)


Structure and functions of kidney & juxta glomerular apparatus, role of
reninangiotensin system ; Renal blood flow; Mechanism of urine formation,
concentration and diluting mechanism; Concept and significance of ‘clearance’
tests; Renal regulation of fluid and electrolytes & acid base balance; Structure and
innervations of urinary bladder, physiology of micturition, cystometry, and its
abnormalities; Artificial kidney(dialysis) and renal transplantation; Renal Function
Tests.

Endocrine Physiology: (PY 8.1 – 8.6) (15 hrs)


Mechanism of action of steroid, protein and amine hormones; Synthesis,

55
secretion, transport, physiological actions, regulation and effect of altered (hypo
and hyper) secretion of pituitary gland, thyroid gland, parathyroid gland, adrenal
gland, pancreas and hypothalamus; Physiology of bone and calcium metabolism;
Physiology of growth; Physiology of Thymus &Pineal Gl and; Hormone function
tests; Obesity & metabolic syndrome; Stress response
Reproductive Physiology: (PY 9.1 – 9.12) (10 hrs)
Sex determination; sex differentiation and their abnormalities; Puberty:
onset, progression, stages; early and delayed puberty; Male reproductive system:
functions of testis, spermatogenesis and its regulation, Cryptorchidism; Female
reproductive system: functions of ovary and its control, men strual cycle:
Hormonal, uterine and ovarian changes; Tests for ovulation; Physiological effects
of sex hormones; Contraceptive methods for male and female; Effects of removal
of gonads on physiological functions; Physiology of pregnancy, fetoplacental unit,
pregnancy tests, parturition & lactation; Semen analysis; Causes and principles of
management of infertility; Hormonal changes and their effects during per
menopause and menopause; Psychological and psychiatric disturbances associated
with reproductive physiology.
Neurophysiology: (PY 10.110.20) (35 hrs)
Organization of nervous system; Sensory system: types, functions and properties of
synapse, receptors, reflex; Somatic sensations & sensory tracts; Physiology of pain;
Motor system: organization, motor tracts, mechanism of maintenance of tone,
control of voluntary movements ; Posture and equilibrium & vestibular apparatus;
Reticular activating system, Autonomic nervous system ; Spinal cord: functional
organization and lesions ; Formation, circulation and functi on of CSF; Blood brain
barrier; Neurotransmitters. Organization, connections and functions of cerebral
cortex, basal ganglia, thalamus, hypothalamus, cerebellum and limbic system and
their abnormalities; Higher mental functions ; Physiology of sleep, memory,
learning and speech and their disorders; EEG. Special senses Smell and taste
sensation and their abnormalities; Functional anatomy of ear and auditory
pathways & physiology of hearing, Deafness, hearing tests; Functional anatomy of
eye, Image formation, Visual pathway and its lesions, Physiology of vision including
acuity of vision, colour vision, field of vision, refractive errors, physiology of pupil;
light reflex, accommodation reflex, dark and light adaptation; Auditory & visual

56
evoked potentials.

Integrated Physiology: (PY 11.1 – 11.14) (5 hrs)


Temperature regulation: mechanism, adaptation to altered temperature
(heat and cold environment), mechanism of fever, cold injuries and heat stroke;
Exercise cardio respiratory and metabolic adjustments during exercise (isotonic
and isometric), exercise in heat and cold, physical training effects; Physiological
consequences of sedentary lifestyle; Brain death; Physiology of Infancy*;
Physiology of aging free radicals and antioxidants*; Physiology of meditation*.

(* ‘Noncore’ competencies as per “Competency based Undergraduate


Curriculum for the Indian Medical Graduate 2018: Medical Council of India”).

Time table is planed tentatively for 45 weeks (with seven weeks of vacation +
public holidays + term assessments i.e. 52-7= 45 weeks) so theory/practical’s/tutorials
and SGT/SDL/ – 495 hours.
ECE – 30 Hours total - 495+30=525hours.

57
PRACTIALS

The following list of practical is minimum and essential. Additional exercises


can be included as and when feasible and required. All the practicals have been
categorized as ʹProcedures to be performed’ and ʹDemonstrationsʹ.The
procedures are to be performed by the students during practical classes to acquire
skills. These would be included in the practical during University examination.
Those categorized as ‘Demonstrations’ are to be shown to students during
practical classes. Questions based on these would be given in the form of data,
charts, graphs, problems and case histories for interpretation by students during
university examination.

I. Procedures to be performed by the students:


a. Hematology:

1. RB Ccount
2. WBC Count
3. Differential Leuco cyte Count
4. Estimation of hemoglobin
5. Blood grouping
6. Bleeding time
7. Clotting (Demonstration of fibrin thread)
8. Calculate RBC indices MCV, MCH,MCHC.
b. Procedures to be performed on Human subjects:
1. Mosso’s ergography.
2. Recording of Blood Pressure, Pulse rate at rest and effect of posture.
3. Effect of mild and moderate exercise on blood pressure, pulse rate
and respiratory rate.
4. Demonstrate Harvard step test and describe the impact on induced
physiologic parameters.
5. Record and interpret Lead II ECG. Given a normal ECG, determine cardiac
axis.
6. Spirometry Lung volumes and capacities, MVV and Dyspnoeic Index,
Timed vital capacity.
7. Peak Expiratory flow rate.

58
8. Demonstrate Basic Life support in a simulated environment.
9. Visual field by Perimetry.
c. Clinical Examination:
1. Components of history taking and general physical examination
2. Examination of radial pulse
3. Examination of Cardiovascular system
4. Examination of Respiratory system
5. Examination of abdomen
6. Examination of Higher mental functions
7. Examination of Sensory system
8. Examination of Motor system including reflexes.
9. Examination of Cranial Nerves
II. Demons trations:

1. Hematology:
1. Erythrocyte sedimentation rate
2. Haematocrit
3. Reticulocyte count
4. Platelet count
5. Osmotic fragility
2. Record Arterial pulse tracing using finger plethys mography*
3. Stethography
4. Tests of cardiovascular autonomic functions*

(* ‘Non‐core’ competencies as per “Competency based


Undergraduate Curriculum for the Indian Medical Graduate 2018:
Medical Council of India”)

III. Interpretation:

Charts, Problems, Graphs and Casehistories.


Chart also includes Interpret growth chart*, Interpret anthropometric
assessment of infants*: (*these two charts are ‘Noncore’ competencies
as per “Competency-based Undergraduate Curriculum for the Indian
Medical Graduate 2018: Medical Council of India”)

59
IV.

(i) Amphibian nerve-muscle experiments and interpretation ofgraphs

List of graphs on nerve-muscle experiments:


 Simple muscle twitch
 Effect of various strengths of stimuli on Simple muscle twitch
 Effect of changes in temperature on Simple muscle twitch
 Effect of two successive stimuli on muscle contraction
 Effect of multiple successive stimuli (treppe, clonus, tetanus)
 Study of fatigue in skeletal muscle
 Effect of load on muscle

(ii) Amphibian cardiac experiments and interpretation of graphs

List of graphs on cardiac experiments:


 Normal cardiogram
 Effect of temperature on frog heart
 Effect of Stanniusligatures
 Properties of cardiac muscle – all or none law, staircase effect, the
refractory period in a beating heart(extra systole and compensatory
pause),refractory period in aquiescent heart.

60
SUGGESTED TEXT BOOKS
Note: A single text book may not cover the entire curriculum. Referring to
more than one book is recommended.

TEXT BOOKS (latest editions)

1. Guyton and Hall. Text of Medical Physiology. South Asian edition. Mario

Vaz, Anura Kurpad, TonyRaj.


2. Ganong’s Review of MedicalPhysiology.

3. Text book of Medical Physiology. InduKhurana

4. Text book of Medical Physiology. D Venkatesh, H HSudhakar

5. Comprehensive text book of medical physiology. G K Pal. – singlevolume


6. Berne and Levy Physiology. BM Koeppen, BAStanton

7. Text book of physiology –A.K..JAIN

8. Text book physiology by chaudhari

Reference books for practicals:


1. Hutchison’s ClinicalMethods.
2. Text book of practical physiology. GK Pal and PravatiPal
3. A textbook of Practical Physiology. CL Ghai
4. Text book of practical physiology by varshini
5. Text book of practical physiology by A.K.Jain

61
Weightage of marks in physiology university theory examinations

Paper – I Max 100 Marks

Topics Marks Allocated

General Physiology 05

Hematology 20

Cardiovascular Physiology 25

Respiratory Physiology 20

Gastrointestinal Physiology 15

Renal Physiology 15

Paper – II
Max 100 Marks
Topics Marks Allocated

Nerve and Muscle Physiology 15

Endocrine physiology 20

Reproductive physiology 15

Central nervous system 35

Special senses 10

Integrated Physiology 05

62
PHYSIOLOGY PAPER I

DATE: TIME: 3 HOURS


Answer all questions MAX MARKS: 100
Draw diagrams whenever necessary

SECTION I

MULTIPLE CHOICE QUESTIONS (20 x 1 = 20 marks)

SECTION II

ESSAY QUESTIONS - 15 MARKS EACH (2 x 15 = 30 marks)

1. What is Hypoxia? Classify it. Explain the mechanism and add a note on oxygen
therapy. (2+2+8+3=15)
2. Define cardiac output. Explain briefly about regulation of cardiac output. Write the
factors affecting cardiac output and methods of estimation. (2+7+3+3)

SHORT QUESTIONS - 5 MARKS EACH (10 x 5 = 50 marks)

3. Juxta glomerular apparatus


4. Deglutition
5. Describe mechanism of coagulation
6. Homeostasis
7. Micturition reflex
8. Dysbarism
9. A 35-year-old business executive complaints of pain in epigastric region which is
relived by taking food. His basal secretion of HCL has 6 meq/L during augmented
histamine test was 35 meq/L
a. What is your likely diagnosis?
b. What is the normal value of Augmented Histamine test?
c. Mention other reasons that can be used to provoke gastric secretion
d. Why pain is relieved after food
e. What will be the effect of vagotomy in this patient?
10. Short term regulation of blood pressure
11. Cell mediated immunity
12. A 25-year-old female complaints of dizziness, fatigue, difficulty in concentration and
sometimes with palpitation. On examination patient showed pallor of skin, mucous
membrane, spooning of nails. Hematological investigations revealed Hb – 6 gms %,
RBC count – 3 millions / cu.mm, MCHC: 28 gms /100ml /c.mm of blood, MCV: 60
cubic microns. a) What is the probable diagnosis? b) What is the probable cause?
c) What is the treatment?

63
PHYSIOLOGY PAPER II

DATE: TIME: 3 HOURS


Answer all questions MAX MARKS: 100
Draw diagrams whenever necessary

SECTION I

I. MULTIPLE CHOICE QUESTIONS (20 x 1 = 20 marks)

SECTION II

II. STRUCTURED ESSAY QUESTIONS - 15 MARKS EACH (2 x 15 = 30 marks)

1. Describe the origin, course, termination of corticospinal tract with a diagram. Explain
its functions (2+5+2+4+2 =15 Marks)
2. Describe the synthesis and secretion of thyroxine. Discuss the mechanism of actions
and regulation of secretion of thyroxine. List out the actions of thyroxine. Add a note
on Grave’s disease. (2+6+2+2+2+1)

III. SHORT ESSAY QUESTIONS - 5 MARKS EACH (10 x 5 = 50 marks)

3. Connections and functions of basal ganglia


4. Visual pathway and its lesions
5. Menstrual cycle
6. Hyperthermia
7. Organ of Corti
8. Contraceptives
9. A 40-year-old male came with a drunken gait, getting shakes in hands while doing
actions. On examination, intentional tremors seen and coordination is lost during both
closed/opened eyes.
a. What is your provisional diagnosis?
b. What is the dysfunction due to?
c. what are all the tests we can do to confirm the diagnosis.
10. Neuromuscular junction
11. Brown Sequard syndrome
12. A female patient 40 years age complaints of drooping of eyelid, general weakness and
fatigue that aggravates during evening but improves after rest.
a. What is your provisional diagnosis?
b. What is the dysfunction due to?
c. Give the physiological basis of use of drug in this condition

64
BIOCHEMISTRY
The course will comprise Molecular and Cellular Biochemistry.
(a) Competencies: The learner must demonstrate an understanding of:
1. Biochemical and molecular processes involved in health and disease,
2. Importance of nutrition in health and disease,
3. Biochemical basis and rationale of clinical laboratory tests, and demonstrate ability
to interpret these in the clinical context.
(b) Integration: The teaching/learning programme should be integrated horizontally and
vertically, as much as possible, to enable learners to make clinical correlations and to
acquire an understanding of the cellular and molecular basis of health and disease.

GOAL

The goal is to teach Biochemistry to undergraduate students to make them


understand the scientific basis of life at the molecular level and to orient them
towards the application of the knowledge acquired in solving clinical problems.

OBJECTIVES

A. KNOWLEDGE

At the end of the course, the student should be able to:

1. Describe the molecular and functional organization of a cell and its sub cellular
organ cells
2. Define at structure, function, and inter relationships of biomolecules and their
relation with functional aspects
3. Summarize the fundamental points of enzymology and clinical application.
4. Describe the digestion and assimilation of nutrients and consequences of
malabsorption.
5. Integrate the various aspects of metabolism and their regulatory pathways;
6. Explain the biochemical basis of inherited disorders with their associated
sequelae
7. Describe mechanisms involved in body fluid and PH Homeostasis;
8. Outline the molecular mechanisms of gene expression and
regulation, the principles of genetic engineering and their
application in medicine
9. Summarize the concepts of body defense
10. Outline the biochemical basis of environmental health hazards, biochemical
basis of cancer and carcinogenesis
11. Familiarize with the principles of various conventional and specialized
laboratory investigations and interpretation of a given data

65
B. SKILLS:

At the end of the course, the student should be able to:


1. Use of conventional techniques/instruments to perform biochemical analysis
relevant to screening and diagnosis of disorders
2. Analyze and interpret investigative data
3. Demonstrate the skills of solving scientific and clinical problems and decision-
making

66
Teaching hours

Sl No. Teaching-learning methods No. of Hours

1 Large group teaching 80 hours

2 Small-Group Teaching (SGT) 150 hours


(Small Group Discussions‐SGD/Tutorials/Seminars/Case-based
learning sessions/Integrated teaching sessions/Practical)

3 Self‐Directed Learning (SDL) 20 hours

TOTAL 250 hours

4 Early clinical Exposure 30 hours

69
Syllabus
Sl Topic Compe Hrs
No
tency
1 Importance of Biochemistry in health and disease - 1
2 Cell BI 1.1 3
Organelles, Cell membrane, Transport across cell membranes
3 Enzymes BI 2.1, 5
Definition, General properties, IUB Classification. BI 2.3,
Coenzymes and Cofactors, BI 2.4,
BI 2.5,
Mechanism of Enzyme action, Factors affecting enzyme activity, BI 2.6,
BI 2.7
Enzyme specificity,
Enzyme inhibition and its clinical importance, Isoenzymes –
Definition, Diagnostic Importance of isoenzymes with examples,
Diagnostic Importance of enzymes – LDH, CK, AST, ALT,ALP, GGT,
Amylase, Lipase, G6PD, Cholinesterase, Aldolase, ACP,
5’nucleotidase,
Enzymes as Therapeutic agents.
4 Chemistry of Carbohydrates BI 3.1 3
Definition, Biomedical importance, Classification with examples
Mono saccharides and its derivatives
Disaccharides, oligosaccharides Polysaccharides–Homo
polysaccharides, Hetero polysaccharides Concept of glycation and
glycosylation
Importance of Glycoproteins
Stereoisomerism

70
5 Carbohydrate metabolism BI 3.2, 10
Digestion and absorption, Lactose intolerance, Glucose BI 3.3,
transporters PATHWAYS – Significance, Site, reactions, key steps, BI 3.4,
energetics, regulation, inhibitors and associated disorders BI 3.5,
BI 3.6,
of Glycolysis, Rapaport Leubering cycle and its BI 3.7,
significance Citric acid cycle - Amphibolic role, Anaplerotic BI 3.9
reactions Gluconeogenesis, Cori’scycle
GlycogenesisGlycogenolysis, Glycogen storage disorders
Significance of HMP shunt pathway Galactose
metabolism Fructose metabolism Regulation of blood
glucose in well fed condition and fasting.

6 Chemistry of lipids BI4.1, 3


Definition, Modified Bloor’s classification with examples, BI11.24
Biomedical importance of lipids Fatty acids –
types and importance Triacylglycerol –
composition and importance
Phospholipids - Types, functions with clinical importance Glycolipids
– Types and importance
Cholesterol - structure and biological importance
Lipoproteins - Types and functions
Amphipathic lipids - Definition, examples and importance,
Liposomes
7 Lipid metabolism BI4.2, 10
Digestion and Absorption of lipids BI4.3,
PATHWAYS – Significance, Site, reactions, key steps, energetics, BI4.4,
regulation, and associated disorders of BI4.6
Betaoxidation
Other types of Oxidation of fatty acids
Ketogenesis, ketolysis

71
Cholesterol metabolism
Lipoprotein metabolism-
Hyperlipoproteinemias
Formation and functions of bile acids and bile salts
metabolism of triacylglycerols and Phospholipids.
Lipid storage disorders,
Prostaglandins – types and biomedical importance,
Fatty liver and lipotropic factors

8 Chemistry of aminoacids and Proteins Aminoacids–Classification BI5.1, 3


based on side chain, nutritional requirement and metabolic fate BI5.2
Biologically importantpeptides
Proteins–Definition, Classification based on chemicalnature,
functions and nutritionalvalue
Structural organization of proteins
Denaturation

9 Protein and amino acid metabolism BI5.3, 10


Digestion and absorption and associated BI5.4,
disorders Amino acid pool BI5.5,
General reactions– Transamination, BI11.17
Transmethylation,Transdeamination, Deamination
their significance
Sources and fate of ammonia - Trapping, Transport and Disposalof
ammonia, ammoniatoxicity
Urea cycle and itsdisorders
Glycine – specialised products and their
importance Phenylalanine & Tyrosine metabolism
Tryptophan metabolism
Sulphur containing aminoacids– methionine & cysteine
metabolism Arginine metabolism -Formation of Nitric oxide and
its importance

72
Clinical importance of Branched chain amino acids - MSUD Important
functions/products from histidine, Aspartate, Asparagine, glutamate,
glutamine,
Polyamines - Examples and importance
10 Plasma proteins BI5.2 3
Functions and clinical importance of plasma proteins
11 Integration of metabolism BI6.1, 5
Metabolic processes taking place in specific organs in the body in BI3.8,
fed , fasting and exercise states. BI4.5,
Metabolic changes during starvation BI4.7,
Adipose tissue–Hormones secreted from adipose tissue (adipokines– BI3.10,
leptin, adiponectin) their functions and role in hunger and satiety. BI11.17
Diabetes mellitus – types, metabolic changes and complications.
Guidelines for diagnosis of Diabetes mellitus
Dyslipidemia
12 Biological Oxidation BI6.6 4
High Energy Compounds–Definition, Classification, biological
significance.
Electron Transport Chain
Oxidative Phosphorylation
Inhibitors, Uncouplers and their significance.
13 Heme metabolism BI6.11, 6
Heme –Outline of Synthesis, porphyrias Bilirubin BI6.12,
metabolism BI5.2,
Jaundice – definition, types, causes, lab diagnosis BI11.17
Congenital hyperbilirubinemias
Hemoglobin –types
Abnormal hemoglobins
Hemoglobinopathies
14 Extracellularmatrix BI9.1, 3
Composition of ECM–Proteins (Composition and functions of BI9.2

73
Collagen,elastin, fibrillin, fibronectin, laminin) and Proteoglycans.
Involvement of ECM components in health and disease.
Eg.Osteogenesis Imperfecta, Ehler‐Danlos syndrome etc
15 Vitamins BI6.5 11
RDA, Sources, Metabolism, Biochemical functions, Deficiency
manifestations and Hyper vitaminoses of
Fat soluble vitamins(A,D,E,K),
Water soluble vitamins ‐ Vitamin C, Folic acid, Vitamin B12,
Thiamine, riboflavin, Niacin, Pyridoxine, Biotin, Pantothenic
acid
Antivitamins
16 Minerals BI6.9, 5
Major elements and trace elements BI6.10
Sources, RDA, absorption and transport, Homeostasis, Functions,
Biological reference range, disorders associated with –
Calcium, phosphorus, Iron
Functions, clinical importance and disorders associated with
Sodium, Potassium, Copper, Zinc, Selenium, Fluoride, Iodine
and Magnesium
17 Chemistry of Nucleic acids BI7.1 4
Nitrogenous bases: Purines and Pyrimidines
Nucleosides and Nucleotides – Structure, examples, Importance
Synthetic Nucleotide Analogues and their application
Structure and function of DNA (B-DNA)
Structural organization of DNA to form
chromatin
Types of RNA (hnRNA, mRNA, rRNA, tRNA, snRNA) with structure
and functions
miRNA and siRNA - applications in medicine
18 Nucleotide metabolism BI6.2, 6
Sources of atoms of Purine and pyrimidine ring BI6.3,
Salvage pathways of Purine and pyrimidine synthesis Catabolism BI6.4

74
of Purines, Uric acid and its importance
Etiology, manifestations and biochemical basis of clinical manifestations
of– Gout, Lesch-Nyhan syndrome
19 Molecular Biology BI7.1, 10
DNA Metabolism BI7.2,
DNA replication prokaryotic and eukaryotic replication, BI7.3,
requirements, process, inhibitors BI9.3
Telomere, Telomerase and its importance
DNA repair mechanisms
Diseases associated with DNA repair– Eg.XerodermaPigmentosum
Mutations, causes, types of mutation, Consequences with examples
RNA Metabolism
Transcription process in prokaryotes and eukaryotes Post
transcriptional modifications of all types of RNA Protein
Biosynthesis
Genetic Code and itscharacteristics
Translation in Eukaryotes
Posttranslational modifications
Regulation of Geneexpression

20 Molecular biology techniques and Gene therapy Recombinant BI7.4 5


DNA technology, DNA cloning process and application
PCR technique and its application Blotting
techniques
Concept, types and application of gene therapy. DNA
Polymorphism, SNP, VNTR, RFLP
DNA genomic and cDNA libraries DNA Probes DNA
Microarrays
Overview of Human Genome Project HGP

21 Biochemistry of Cancer BI10.1, 4

75
Cell cycle, regulation, abnormal cell growth, programmed cell BI10.2
death (apoptosis)
Mutagens and carcinogens: Definitions, examples and their
actions in carcinogenesis
Proto oncogenes and their activation, oncogenes, tumour
suppressor genes and their role in development of cancer
Oncogenic viruses
Growth factors and their receptors
Tumour markers and their importance in diagnosis and
prognosis of cancer Biochemical basis of cancer therapy
22 Immunology BI10.3 3
Cellular and humoral components of immune system ,
Immunoglobulins – Classes, structure and functional BI10.4
relationship Role of T-helper cells in immune responses ,
Concept of Immune tolerance and BI10.5
Autoimmunity Antigens and concepts in
vaccine development
23 Nutrition BI8.1, 8
Energy content of food items BI8.2,
BMR–Definition, Normal values, Factors affecting and BI8.3,
biomedical importance BI8.4,
SDA – Definition and significance (Thermogenic effect of food) BI8.5,
Nitrogen balance BI11.17,
Balanced diet – definition, composition BI11.23,
Dietary fibres – definition, examples, importance BI11.24
Glycemic index – definition, calculation, importance
Nutritional importance of Carbohydrates, Lipids ,Proteins,
Vitamins and minerals,
Nutritional indices
Calculation of calorie requirement
Dietary advice for optimal health in childhood and adults,
special conditions like diabetes, coronary artery disease,
pregnancy.

76
Types, causes and effects of Protein energy malnutrition Obesity–
Definition, BMI, types, causes, role of GI peptides and
adipokines in obesity, associated health risks
eg.,metabolic syndrome
24 Organ function tests BI6.13, 4
Liver Function Tests BI6.14,
Renal Function tests BI6.15,
Thyroid function tests BI11.17
Adrenal function
tests
25 Acid base balance BI6.7, 5
Concept of Acids,Bases and buffers, HH Equation and its BI6.8,
application BI11.17
Regulation of pH by buffers, respiratory andrenal mechanisms
Anion gap and itssignificance
Acidosis and alkalosis (metabolic andrespiratory)–causes,
compensatory mechanisms and lab findings

77
26 Water and electrolyte balance BI6.7 2
Distribution of water and electrolytes in ICF and ECF Osmolality
of ECF
Regulation of water and electrolyte balance
Disorders of electrolyte imbalance–causes and clinical features of
Hyperkalemia, Hypokalemia, Hypernatremia, Hyponatremia

27 Free Radicals and Antioxidants BI7.6, 3


Free radicals, Reactive oxygen species(ROS), Reactive nitrogen BI7.7
species(RNS) Damaging effects of ROS on bio molecules, lipid
peroxidation
Antioxidant defence system of our body–enzymes, vitamins,
metabolites as antioxidants
Role of oxidative stress in atherosclerosis, diabetes mellitus and
cancer

28 Xenobiotics andDetoxification BI7.5 3


Biotransformation – phase I &II

29 Clinical chemistry BI11.16 3


Basic concepts of clinical chemistry
Biological reference intervals

78
RECOMMENDEDBOOKS

TEXTBOOKS: (Recent editions)

1. Harpers’ Illustrated Biochemistry


2. Lippincotts’ Illustrated reviews –Biochemistry
3. DM Vasudevan. Textbook of Biochemistry for Medical students
4.S.K.Gupta. Biochemistry for MBBS
5.Pankaja Naik.Biochemistry
6.Dinesh Puri. Textbook of Biochemistry
7.Namrata Chhabra. Case oriented approach towards Biochemistry
8. MDRafi. Textbook of Biochemistry

REFERENCE BOOKS: (Recent editions)

1. Marshall and Bangert. Clinical Chemistry


2. Stryer. Biochemistry
3. Lehninger: Principles of Biochemistry
4. Varley's Practical Clinical Biochemistry

79
Practicals

1: Qualitative Experiments

a) Analysis of Normal constituents of urine BI11.3, BI11.4 -DOAP


b) Analysis of Pathological Constituents of Urine BI11.4, BI11.20 -DOAP

2: Quantitative Experiments

a) Estimation of plasma glucose BI11.21, BI3.10 –DOAP


b) Estimation of serum and urine creatinine,Creatinine clearance
BI11.7,BI11.21-DOAP
c) Estimation of blood urea BI11.21-DOAP
d) Estimation of Total Protein and Albuminin serum,A:GratioBI11.8,BI11.21-
DOAP
e) Estimation of Total chole sterol and High density lipo protein (HDL)
cholesterolBI11.9-DOAP
f) Estimation of Triacylglycerols BI11.10- DOAP
g) Estimation of Calcium and Phosphorous BI11.11-DOAP
h) Estimation of Serum Bilirubin BI11.12-DOAP
i) Estimation of AST, ALT activity BI11.13-DOAP
j) Estimation of AL Pactivity BI11.14-DOAP

3: Demonstrations
a) Lab safety and Biomedical waste disposal, Commonly used lab equipment,
glassware, and reagents BI11.1
b) Preparation of buffers and estimation of pH using pH meter
BI11.2,11.16,11.19
c) Colorimetry, Spectrophotometry BI11.6,BI11.18
d) Specimen collection and pre analytical errors in the clinical Biochemistry
lab
e) Clinical chemistry auto analyzer and quality control (Internal and
External quality control, Precision, Accuracy, QC
rules),BiologicalreferenceintervalsBI11.16,BI11.19
f) Serum protein electrophoresis, types and applications BI11.16,BI11.19
g) Paper chromatography/ TLC of amino acids/ sugars, types and applications
BI11.5,BI5.5,BI11.16,BI11.19
h) Estimation of serum electrolytes by ISEBI11.16,BI11.19
i) Blood gas analysis using ABG analyzer BI11.16,BI11.19
j) The principle, procedure, and applications of ELISA, protein extraction,
Blotting techniques, PAGEBI11.16,BI11.19
80
k) Principle, procedure and applications of PCR, DNA isolation
BI11.16,BI11.19

4: Discussion and interpretation


a) Calculate energy content in different food items BI11.23
b) Use of unsaturated, saturated and trans fat in food BI11.24Composition of
CSF BI11.15

CERTIFICATION OF SKILL ACQUISITION:

Sl Competency to be certified with Competency number


No
1 Perform urine analysis to determine normal constituents (BI11.4)

2 Identify abnormal constituents in urine, interpret the findings and correlate


these with pathological states (BI11.20)

3 Demonstrate estimation of glucose in serum (BI11.21)


4 Demonstrate the estimation of serum Creatinine and Creatinine clearance(BI11.7,
BI11.21)
5 Demonstrate the estimation of urea in serum (BI11.21)
6. Demonstrate estimation of serum protein, albumin and A:G ratio (BI11.7,BI11.21)

81
Weightage of marks in biochemistry university theory examinations
Paper – I
TOPIC Upto
Marks
Enzymes 8
Biological Oxidation 8
Digestion and absorption 3
Detoxific 3
ation
Carbohydrate chemistry and metabolism 16

Vitamins 15
Free Radicals and Antioxidants 3
Nutrition 8
Lipid chemistry and metabolism 15
Heme metabolism 10
Organ Function tests 8
Integration of metabolism and
homeostasis 3
TOTAL 100

Paper-II
TOPIC Upto
Marks
Protein chemistry and metabolism 15
Mineral metabolism 15
Nucleic acid chemistryand metabolism 8

Molecular Biology 15
Hormone s 5

Extracellular matrix 5
Plasma proteins 6
Immunol 5
ogy
Cell and organelles, Cell membrane, 3
Transport across cell membranes

Cancer 8
Acid-Base balance and water-
Electrolyte balance 15

TOTAL 10
82 0
Weightage of marks(approximate) in biochemistry university
theory examinations

Paper – I
TOPIC APPROXIMATE MARKS

Enzymes 10
Biological Oxidation 5
Digestion and absorption 5
Detoxification 5
Carbohydrate chemistry and metabolism 15
Vitamins 15
Free Radicals and Antioxidants 5
Nutrition 10
Lipid chemistry and metabolism 15
Heme metabolism 10
Organ Function tests 5
Integration of metabolism and Homeostasis 5

Paper-II
TOPIC APPROXIMATE
Protein chemistry and metabolism MARKS 15
Mineral metabolism 15
Nucleic acid chemistry and metabolism 10
Molecular Biology 15
Hormones 5
Extracellular matrix 5
Plasma proteins 5
Immunology 5
Cell and organelles, Cell membrane,
Transport across cell membranes 5
Cancer 5
Acid-Base balance and water-
Electrolyte balance 15

83
Biochemistry Model Paper I
Max Marks: 100 Time: 3hrs

I. MCQs = 1x20=20 M

II. Structured Essay Questions: 2x15= 30 M

1. Write the sources & RDA of Vitamin D. Describe the synthesis of active form of Vitamin D.
Explain the biochemical functions and deficiency manifestations of Vitamin D. Add a
note on hypervitaminosis D. (1+1+4+4+3+2)

2. Name Lipoproteins and mention their functions. Explain the LDL metabolism. What is normal
serum cholesterol level? What are the derivatives of cholesterol and add a note
on atherosclerosis. (2+2+4+1+3+3)

III. Short Essay Questions: 10x5=50 M

3. A 5-year-old boy was admitted to a medical ward in a comatose condition. The boy
maintained good health till very recently. His father said that the boy complained of thirst and
increased urination. He lost weight and became very thin. His breath had fruity odor. His lab
report as follows
Blood glucose : 800mg/dL, Blood urea: 40mg/dl, Serum creatinine : 1.8mg/dl
Benedicts Test : ++++ , Rothera’s test : ++++
a) What is the probable diagnosis in the above case
b) What are the point’s in favour of such a diagnosis?
c) What is the reason for the fruity odour
d) Name two non-sugar compounds that give a positive Benedicts test

4. Enumerate the types of Heteropolysacharides and write their functions


5. Explain Competitive Inhibition and give 2 examples
6. Describe the types of Haemoglobinopathies
7. Describe Chemiosmotic theory and give two examples for uncouplers
8. Describe the types of Protein Energy Malnutrition
9. Describe the steps in Heme synthesis
10. Describe the Liver Function Tests
11. A 4-year-old boy came to the hospital and his serum ALP levels are high.
a) Enumerate physiological and pathological conditions for raised ALP in this age group.
b) List the Isoenzymes of ALP.
c) Reference range of serum ALP

12. Metabolic adaptation during starvation

84
Biochemistry Paper II

Max Marks: 100 Time: 3hrs


I. MCQs =1x20=20M

II. Structured Essay Questions:2x15=30M

1) Write the sources & RDA of Calcium. Describe the functions of calcium. Explain the
regulation of serum calcium level. What is normal serum calcium level and add a note
on deficiency manifestations. (1+1+5+5+1+2)

2) What is the normal blood pH. Describe the buffer mechanisms regulating blood pH.
Explain about metabolic acidosis and add a note on Anion Gap. (1+8+4+2)

III. Short Essay Questions: 10x5=50 M

3) A three-month-old child was brought to the hospital by the parent with the
complaint of the child’s diapers becoming black.
a) Which metabolic pathway is affected?
b) what is the biochemical cause behind the black diapers.
c) This analyte can give a false positive with which urine test?
d) What complication can be expected in middle age in this child due to this
compound?

4) A 50-year-old alcoholic man came with the complaints of excruciating pain in his great
toe. He gave a history of taking a heavy non-vegetarian meal and alcohol the night
before. What is your diagnosis. Enumerate the primary and secondary causes of the
disease. What is the drug of choice for this condition?

5) Define and describe the different types of mutations

6) Describe the mechanism of action of Group I hormones

7) Describe the structure of collagen

8) Discuss the steps of PCR and its applications

9) Describe the types of acquired immunity and their functions

10) Enumerate the tumor markers and their role in cancer diagnosis

11) Describe the synthesis of melanin

12) Explain the transport mechanisms across the cell membrane

85

You might also like